Showing a limit of the recurrence relation

Click For Summary
The recurrence relation defined by x_{n+1} = 3 x_n^2 with x_1=1 leads to the conclusion that if a limit exists, it must be either 0 or 1/3. However, the sequence is unbounded, indicating that the limit does not actually exist. The discussion highlights that the convergence of the sequence depends on the initial value; if |x_1| < 1/3, it converges to 0, if |x_1| = 1/3, it converges to 1/3, and if |x_1| > 1/3, it diverges. The possibility of a limit approaching infinity is also noted, emphasizing the complexity of the problem.
Mr Davis 97
Messages
1,461
Reaction score
44

Homework Statement


Let ##x_1=1## and ##\displaystyle x_{n+1} = 3 x_n^2## for ##n \ge 1##.
a) Show if ##a = \lim x_n##, then ##a = 1/3## or ##a = 0##.
b) Does ##\lim x_n## actually exist?

Homework Equations

The Attempt at a Solution


I have proven before that, in general, ##\lim s_{n+1} = \lim s_n##. Hence to answer a), we simply take the limit of both sides and solve the quadratic equation. Easy. But clearly, since ##x_n## is not bounded, the limit doesn't actually exist. Why am I getting values ##1/3## or ##0## when the limit doesn't actually exist?
 
Physics news on Phys.org
What happens if ##x_1=1/3##? What happens if ##x_1<1/3##?
 
I found the ##a=0## case, but not the ##a=\frac{1}{3}##. Anyway, if it this editor here allowed me to write letters upside down, I will win in the lottery tomorrow. That's 100% certain and true. What changed between a) and b) and have you used all information given for a) same as for b)?
 
I think I see... In part a), are we saying that if ##x_n## converges, one of those would be the resulting value. In part b), I guess the question of whether the sequence actually converges is dependent on the initial value, as if ##|x_1| < 1/3## it converges to 0, ##|x_1| = 1/3## it converges to 1/3, and if ##|x_1| > 1/3## it diverges.
 
Mr Davis 97 said:
I think I see... In part a), are we saying that if ##x_n## converges, one of those would be the resulting value. In part b), I guess the question of whether the sequence actually converges is dependent on the initial value, as if ##|x_1| < 1/3## it converges to 0, ##|x_1| = 1/3## it converges to 1/3, and if ##|x_1| > 1/3## it diverges.
No, in b) you used ##x_1=1## to see that it does not converge. In a) you reasoned by ##\lim s_n = \lim s_{n+1}##. Where's the initial value here? It is the other way around. And in a) we have if then, but if is never true.
 
  • Like
Likes Mr Davis 97
Mr Davis 97 said:

Homework Statement


Let ##x_1=1## and ##\displaystyle x_{n+1} = 3 x_n^2## for ##n \ge 1##.
a) Show if ##a = \lim x_n##, then ##a = 1/3## or ##a = 0##.
b) Does ##\lim x_n## actually exist?

Homework Equations

The Attempt at a Solution


I have proven before that, in general, ##\lim s_{n+1} = \lim s_n##. Hence to answer a), we simply take the limit of both sides and solve the quadratic equation. Easy. But clearly, since ##x_n## is not bounded, the limit doesn't actually exist. Why am I getting values ##1/3## or ##0## when the limit doesn't actually exist?

If the limit exists (call it ##a##) we have ##a = 3 a^2##, so ##a = 0## or ##a = 1/3##.

However, what if the limit does not exist? In that case, a third solution to ##a = 3 a^2## "exists" within the extended real-number system, namely: ##a = +\infty##.

What do you think is the correct value of ##a## in this problem?
 
Question: A clock's minute hand has length 4 and its hour hand has length 3. What is the distance between the tips at the moment when it is increasing most rapidly?(Putnam Exam Question) Answer: Making assumption that both the hands moves at constant angular velocities, the answer is ## \sqrt{7} .## But don't you think this assumption is somewhat doubtful and wrong?

Similar threads

  • · Replies 13 ·
Replies
13
Views
4K
  • · Replies 1 ·
Replies
1
Views
2K
  • · Replies 3 ·
Replies
3
Views
2K
Replies
11
Views
2K
  • · Replies 9 ·
Replies
9
Views
2K
  • · Replies 11 ·
Replies
11
Views
2K
  • · Replies 1 ·
Replies
1
Views
2K
  • · Replies 1 ·
Replies
1
Views
2K
  • · Replies 2 ·
Replies
2
Views
2K
Replies
9
Views
2K